What value of x will make the triangle congruent

What Value Of X Will Make The Triangle Congruent

Answers

Answer 1

Answer:

x = 3/5 = 0.6

Step-by-step explanation:

By the ASA Triangle Congruence Theorem, the two given triangles are congruent (vertical angles are congruent). Because we know that corresponding parts of congruent triangles are congruent, we can write the following equation: 7x - 3 = 2x. Now we can solve:

7x - 3 = 2x

-3 = -5x

3/5 = x


Related Questions

Approximately 70% of U.S. adults had at least one pet as a child. We randomly survey 60 U. S. adults. We are interested in the number that had at least one pet as a child. The probability that at least 3 adults had at least one pet as a child means:
A. P(X=0)+P(X=1)+P(X=2)+P(X=3)
B. P(X=0)+P(X=1)+P(X=2)
C. P(X=4)+P(X=5)+P(X=6)+ ...
D. P(X=3)+P(X=4)+P(X=5)+ ...

Answers

Answer:

D. P(X=3)+P(X=4)+P(X=5)+ ...

Step-by-step explanation:

Given

[tex]n =60[/tex]

[tex]pr = 70\%[/tex] -- proportion of adults with pet

Required

Represent at least 3 adult with pet as a probability

At least 3 means 3 or more than.

So, the probability is represented as:

[tex]P(x \ge 3) = P(3) + P(4) + P(5) + ........[/tex]

Hence;

(d) is correct

determine whether the variable is qualitative or quantitative
street name and adress

Answers

Answer:

C

Step-by-step explanation:

Street name has nothing to do with numerical values, therefore it cannot be quantitative. If we take the two qualitative answers left, one of them says something about numbers, so it cannot be that one. C is the only one left over after process of elimination.

f(2) 42 g(x) = 2x +3 Find 4) (. Include any restrictions on the domain. O A. 2.2---3 417 2 > 0 B. 47 2 2 r--3, (1) (a) = (4) (a) x A 을 c. (5) (r) = 1,2*0 OD (1) (a= + 2​

Answers

Answer:

Option D

Step-by-step explanation:

Given f(x) = [tex]\sqrt[3]{4x}[/tex]

g(x) = 2x + 3

Since, [tex](\frac{f}{g})(x)=\frac{f(x)}{g(x)}[/tex]

                     [tex]=\frac{\sqrt[3]{4x}}{2x+3}[/tex]

This function is defined for the denominator is not equal to zero.

(2x + 3) ≠ 0

x ≠ [tex]-\frac{3}{2}[/tex]

Therefore, Option D will be the correct option.

i need help with this!

Answers

9514 1404 393

Answer:

  64.6

Step-by-step explanation:

One standard deviation is 4.3. Then +2 standard deviations is ...

  (+2)(4.3) = +8.6

This amount added to the mean gives ...

  56 +8.6 = 64.6 . . . . +2σ from the mean

Factorize (2ax-2ay-3by+3bx)​

Answers

Answer:

That's the answer I searched for and I hope it can help.

#Carryonlearning

Please make me brainliest!

hope this helps! factorise out the common factor first! feel free to clarify ya!

What is the result when 16x + 9 is subtracted from 4x + 6

Answers

Answer:

(4x+9)-(16x+9) = 4x+9-16x-9

= -12x

Calculating area of rectangle.

Your measurements Another student's measurements
Length (cm) 20.70 20.74
Width (cm) 10.44 10.46
Area (cm2) ________ ________

Required:
Why might two students have different calculated areas when measuring the same rectangle?

Answers

Answer:

Your measurements; Area = 216.108 cm²

Another student's measurements; Area = 216.9404 cm²

- Difference in area could be as a result of human error or perhaps that they made use of different measuring tools.

Step-by-step explanation:

For Your measurements;

Length of rectangle = 20.70 cm

Width of rectangle = 10.44 cm

Area of rectangle is given by; A = length × width = 20.7 × 10.44 = 216.108 cm²

For Another student's measurements;

Length of rectangle = 20.74 cm

Width of rectangle = 10.46 cm

Area = 20.74 × 10.46

Area = 216.9404 cm²

The areas they both obtained are not of equal values and this could be as a result of human error or perhaps that they used different measuring tools.

x−9≥−12 Express your answer as an inequality.

Answers

Answer:

x ≥ −3

Step-by-step explanation:

Step One : Add 9 to both sides.

x − 9 + 9 ≥ −12 + 9

x ≥ −3

There is no more to be done, that is our answer. x ≥ −3.

Hope this helps, please mark brainliest! :)

HELP ASAP. Find the set.

Answers

Given:

The given sets are:

[tex]U=\{x|x\in N\text{ and }x<10\}[/tex]

[tex]A=\{x|x\text{ is an odd natural number and }x<10\}[/tex]

[tex]B=\{x|x\text{ is an even natural number and }x<10\}[/tex]

[tex]C=\{x|x\in N\text{ and }3<x<5\}[/tex]

To find

The set [tex]B\cap U[/tex].

Solution:

We have,

[tex]U=\{x|x\in N\text{ and }x<10\}[/tex]

[tex]U=\{1,2,3,4,5,6,7,8,9\}[/tex]

[tex]B=\{x|x\text{ is an even natural number and }x<10\}[/tex]

[tex]B=\{2,4,6,8\}[/tex]

Now, [tex]B\cap U[/tex] is the set that contains all the common elements of both sets B and U.

[tex]B\cap U=\{2,4,6,8\}[/tex]

[tex]B\cap U=B[/tex]

Therefore, the correct option is A.

f(x) = -2 ( 1 - 1/4 x ) compute f( -3)

Answers

Answer:

-3 1/2 or -3.5

Step-by-step explanation:

* means multiply

just put 3 where the x is

so

f(x) = -2 ( 1 - 1/4 x )

f( -3) = -2 ( 1 - 1/4 * -3 )

f( -3) = -2 ( 1 3/4 )

f( -3) = - 3 1/2

Circle the answer choice below that does not equal the following:
- 48/16
1) 48/-16
2) -3
3) 3
4) -(48/16)

Answers

Answer:

3

Step-by-step explanation:

-48/16 = 48/16

-48/16 = -3/1 = -3

Given tan theta =9, use trigonometric identities to find the exact value of each of the following:_______
a) sec sqr Theta
b) Cot theta
c) cot (pie/2-Theta)
d) csc sqr theta

Answers

Answer:

[tex](a)\ \sec^2(\theta) = 82[/tex]

[tex](b)\ \cot(\theta) = \frac{1}{9}[/tex]

[tex](c)\ \cot(\frac{\pi}{2} - \theta) = 9[/tex]

[tex](d)\ \csc^2(\theta) = \frac{82}{81}[/tex]

Step-by-step explanation:

Given

[tex]\tan(\theta) = 9[/tex]

Required

Solve (a) to (d)

Using tan formula, we have:

[tex]\tan(\theta) = \frac{Opposite}{Adjacent}[/tex]

This gives:

[tex]\frac{Opposite}{Adjacent} = 9[/tex]

Rewrite as:

[tex]\frac{Opposite}{Adjacent} = \frac{9}{1}[/tex]

Using a unit ratio;

[tex]Opposite = 9; Adjacent = 1[/tex]

Using Pythagoras theorem, we have:

[tex]Hypotenuse^2 = Opposite^2 + Adjacent^2[/tex]

[tex]Hypotenuse^2 = 9^2 + 1^2[/tex]

[tex]Hypotenuse^2 = 81 + 1[/tex]

[tex]Hypotenuse^2 = 82[/tex]

Take square roots of both sides

[tex]Hypotenuse =\sqrt{82}[/tex]

So, we have:

[tex]Opposite = 9; Adjacent = 1[/tex]

[tex]Hypotenuse =\sqrt{82}[/tex]

Solving (a):

[tex]\sec^2(\theta)[/tex]

This is calculated as:

[tex]\sec^2(\theta) = (\sec(\theta))^2[/tex]

[tex]\sec^2(\theta) = (\frac{1}{\cos(\theta)})^2[/tex]

Where:

[tex]\cos(\theta) = \frac{Adjacent}{Hypotenuse}[/tex]

[tex]\cos(\theta) = \frac{1}{\sqrt{82}}[/tex]

So:

[tex]\sec^2(\theta) = (\frac{1}{\cos(\theta)})^2[/tex]

[tex]\sec^2(\theta) = (\frac{1}{\frac{1}{\sqrt{82}}})^2[/tex]

[tex]\sec^2(\theta) = (\sqrt{82})^2[/tex]

[tex]\sec^2(\theta) = 82[/tex]

Solving (b):

[tex]\cot(\theta)[/tex]

This is calculated as:

[tex]\cot(\theta) = \frac{1}{\tan(\theta)}[/tex]

Where:

[tex]\tan(\theta) = 9[/tex] ---- given

So:

[tex]\cot(\theta) = \frac{1}{\tan(\theta)}[/tex]

[tex]\cot(\theta) = \frac{1}{9}[/tex]

Solving (c):

[tex]\cot(\frac{\pi}{2} - \theta)[/tex]

In trigonometry:

[tex]\cot(\frac{\pi}{2} - \theta) = \tan(\theta)[/tex]

Hence:

[tex]\cot(\frac{\pi}{2} - \theta) = 9[/tex]

Solving (d):

[tex]\csc^2(\theta)[/tex]

This is calculated as:

[tex]\csc^2(\theta) = (\csc(\theta))^2[/tex]

[tex]\csc^2(\theta) = (\frac{1}{\sin(\theta)})^2[/tex]

Where:

[tex]\sin(\theta) = \frac{Opposite}{Hypotenuse}[/tex]

[tex]\sin(\theta) = \frac{9}{\sqrt{82}}[/tex]

So:

[tex]\csc^2(\theta) = (\frac{1}{\frac{9}{\sqrt{82}}})^2[/tex]

[tex]\csc^2(\theta) = (\frac{\sqrt{82}}{9})^2[/tex]

[tex]\csc^2(\theta) = \frac{82}{81}[/tex]

What is the domain of the relation given by the set of ordered pairs shown?

{ ( - 3, 2 ), ( - 1, 0 ), ( - 3, 5 ), ( 2, - 3 ) }

List the domain as a set, in increasing order.

Answers

Step-by-step explanation:

the domain = {-3, -1, 2}

_____

The domain, as a set in increasing order, will be {-3, -1, 2}.

What are domain and range?

The domain means all the possible values of x and the range means all the possible values of y.

The set of ordered pairs shown below.

{(-3, 2), (-1, 0), (-3, 5), (2, -3)}

Then the domain, as a set in increasing order, will be

⇒ {-3, -1, 2}

More about the domain and range link is given below.

https://brainly.com/question/12208715

#SPJ2

Each side of a square is (7 + 3x) units. Which is the perimeter of the square?

Answers

If one side is 7 + 3x units then 4 sides of the square is 4 (7 + 3x).

4 (7 + 3x)
28 + 12x

Perimeter = 28 + 12x

Will has a book that he wants to read that book is 36 chapters long if he reads 3 chapters a day how many days will it take Will to read the whole book?​

Answers

Answer:

It will take Will 12 days to read the whole book.

Step-by-step explanation:

36 ÷ 3

= 12

:))

36 ➗ 3 = 12

12 days .........

Below are two parallel lines with a third line intersecting them

Answers

Answer:

hey, there is no attached photo. so I don't know how to help

define prism and itz color​

Answers

Answer:

triangle piece of glass that different colors of light travel through

Provide a formula that can be used to determine whether a point in a circle that is centered about (0,0) with a diameter of 12

Answers

Answer:

x² + y² = 36

Step-by-step explanation:

The formula for the circle is

x² + y² = 36

If the point is on the circle it will equal 36 when plugged into the formula

Point(6, 0)

6² + 0 = 36

36 = 36    on the circle

point (5, 1)

5² + 1² = ?

25 + 1 = 26

26 ≠ 36      not on the circle

Step-by-step explanation:

hhajjbahhahahsshahhshshshshssbhssjdj

10)
X + 80
70°
A) 5
C) -10
B) 8
D) 7

Answers

Answer:

C: x=-10

Step-by-step explanation:

Alternate interior angles are congruent to each other meaning that x+80=70 making x equal to -10. I hope this helped and this is one of my favorite units so post more these questions :)

Given f(x) = 52x, evaluate f(–1), f(0), and f(2). Question 1 options:a. 1/25, 0, 625 B. 25, 1, –25 c. 1/25, 0, 25 d. 1/25, 1, 625

Answers

Answer:

 

1/25, 1, 625

Step-by-step explanation:

The value of the function at f(–1), f(0), and f(2) will be 1/25, 1, 625 thus option (d) is correct.

What is a function?

A certain kind of relationship called a function binds inputs to essentially one output.

In other words, the function is a relationship between variables, and the nature of the relationship defines the function for example y = sinx and y = x +6 like that.

As per the given function,

f(x) = [tex]5^{2x}[/tex]

Put x = -1

f(-1) = 5⁻² = 1/5² = 1/625

Put x = 0

f(0) = 5⁰ = 1

Put x = 2

f(2) = 5²ˣ² = 5⁴ = 625

Hence "The function's value at f(-1), f(0), and f(2) is 1/25, 1, 625".

For more about the function,

brainly.com/question/23712366

#SPJ2

A car manufacturing company produces blue and black cars only. If 54% of 85658 total
production is blue cars, how many black cars are there?

Answers

Answer:

About 39,402.

Step-by-step explanation:

You need to find the 46% of black cars.

The board of directors of a corporation must select a president, a secretary, and a treasurer. In how many possible ways can this be accomplished if there are 25 members on the board of directors

Answers

Answer:

2400 ways

Step-by-step explanation:

Combination has to do with the idea of selection. Here we need to select a president, a secretary, and a treasurer.

This means to select three out of twenty five persons.

From;

nCr = n! / ((n – r)! r!)

n = the number of items.

r = how many items are taken at a time.

Thus we have;

25C3 = 25!/(25 - 3)! 3!

=

1.6× 10^25/1.1×10^21 × 6

= 2400 ways

why mathematical induction is used ?(write a 100 words paragraph)​

Answers

Answer:

Mathematical induction is a method of mathematical proof typically used to establish that a given statement is true for all natural numbers ( non-negitive integers). The simplest and most common form of mathematical induction proves that a statement involving a natural number that holds for all values.

What is the simplified base of the function f(x) = One-fourth (Root Index 3 StartRoot 108 EndRoot) Superscript x?

Answers

Answer:

The base is: [tex]3 \sqrt[3]{4}[/tex]

Step-by-step explanation:

Given

[tex]f(x) = \frac{1}{4}(\sqrt[3]{108})^x[/tex]

Required

The base

Expand 108

[tex]f(x) = \frac{1}{4}(\sqrt[3]{3^3 * 4})^x[/tex]

Rewrite the exponent as:

[tex]f(x) = \frac{1}{4}(3^3 * 4)^\frac{1}{3}^x[/tex]

Expand

[tex]f(x) = \frac{1}{4}(3^3^\frac{1}{3} * 4^\frac{1}{3})^x[/tex]

[tex]f(x) = \frac{1}{4}(3 * 4^\frac{1}{3})^x[/tex]

Rewrite as:

[tex]f(x) = \frac{1}{4}(3 \sqrt[3]{4})^x[/tex]

An exponential function has the following form:

[tex]f(x)=ab^x[/tex]

Where

[tex]b \to base[/tex]

By comparison:

[tex]b =3 \sqrt[3]{4}[/tex]

So, the base is: [tex]3 \sqrt[3]{4}[/tex]

s makes
23. For her backpacking trip, Leslie wants
to carry no more than 20 pounds.
Which inequality represents all the
possible weights w that she is willing
to carry?
AW> 20
BW< 20
CW 220
n
DW = 20

Answers

Answer:

b

Step-by-step explanation:

Note that

> means greater than

< means less than

≥ means greater than or equal to  

≤ less than or equal to  

for example : 2 < 3 means 2 is less than 3

3 >2 means 3 is greater than 2

leslie wants to carry no more than 20 pounds. this means she wants to carry less than 20 pounds

w < 20

Can you help me with this question? 15 points and brainliest if right!!

Answers

Hi um I think you posted a picture of something else other than the question!!

How to take a screenshot on an HP laptop

Press the Windows key and Print Screen at the same time to capture the entire screen. ...

Open an image editing program (Microsoft Paint, GIMP, Photoshop, and PaintShop Pro will all work).

Open a new image and press CTRL + V to paste the screenshot.

please answer correctly thanks​

Answers

a = 0.06
b = 0.08
c = 133

Let f(x)= 6x^2+7x-4. Find:
a) f(3)
b) f(m)
c) f(x+1)

SHOW WORK PLEASE.

Answers

Answer: b) f(m)

Step-by-step explanation:

Adam is 9 years old and Billy is 11 years old. What will be the ratio of Adams’ age to Billy’s age in exactly one year’s time? Give your answer in its simplest form. *

Answers

Answer:

10 / 12

simplified:

5/6

since 5 is a prime number, we can't simplify this ratio any further

Executives from Six Flags, a well-known amusement park chain, had interest in constructing a Six Flags theme park in a location near Ames city limits. Experts believed that approximately 15% of the surrounding population would be interested in becoming season ticket holders. A random sample of 500 residents of Story County was collected (of the approximately 80,000 residents of Story County). Of the 500 people sampled, 124 said that they would be interested in purchasing season tickets to a Six Flags in Ames. The mean of the sampling distribution for the sample proportion when taking samples of size 500 from this population is equal to ___________.

Answers

Answer:

The mean of the sampling distribution for the sample proportion when taking samples of size 500 from this population is equal to 0.248.

Step-by-step explanation:

Central Limit Theorem

The Central Limit Theorem establishes that, for a normally distributed random variable X, with mean [tex]\mu[/tex] and standard deviation [tex]\sigma[/tex], the sampling distribution of the sample means with size n can be approximated to a normal distribution with mean [tex]\mu[/tex] and standard deviation [tex]s = \frac{\sigma}{\sqrt{n}}[/tex].

For a skewed variable, the Central Limit Theorem can also be applied, as long as n is at least 30.

For a proportion p in a sample of size n, the sampling distribution of the sample proportion will be approximately normal with mean [tex]\mu = p[/tex] and standard deviation [tex]s = \sqrt{\frac{p(1-p)}{n}}[/tex]

Of the 500 people sampled, 124 said that they would be interested in purchasing season tickets to a Six Flags in Ames.

This means that [tex]p = \frac{124}{500} = 0.248[/tex]

The mean of the sampling distribution for the sample proportion when taking samples of size 500 from this population is equal to

By the Central Limit Theorem, it is equal to the sample proportion of 0.248.

Other Questions
Determine the mass in grams of hydrogen gas produced if 40.08 g of calcium react with excess aqueous hydrogen phosphate. Marking brainliest Ayuda!!! EllusFind the range of the data.The ages of people eatingchocolate ice cream at a park.12, 2, 8, 5, 3, 2,5Range: [? ]Enter the number that belongs in the green box. Which best describes the electric field created by a positive charge?A. It circles clockwise around the charge.B. It circles counterclockwise around the charge.C. Its rays point away from the charge.D. Its rays point toward the charge. 3y - 2 = 13 solve for y Select 3 sides that can form a right triangle. Which value of n would make 3n=8 Brainliest goes to whoever answers correctly I have other questions if you want more points Consider the following argument from analogy. According our rules for appraising analogical reasoning, if a subsequent consideration strengthens the argument, answer a. Answer b if a consideration weakens the argument. Answer c if a consideration does not affect the argument.Bill has taken three history courses and found them very stimulating and valuable. So he signs up for another one, confidently expecting that it too will be worthwhile.25. Suppose that his previous history courses were in ancient history, modern European history, and American history.26. Suppose that his previous history courses had all been taught by the same professor that is scheduled to teach the present one.27. Suppose that his previous history courses all had been taught by Professor Smith, and the present one is taught by professor Jones.28. Suppose that Bill had found his three previous history courses to be the most exciting intellectual experiences of his life.29. Suppose that his previous history courses had all met at 9:00am, and the present one is scheduled to meet at 9:00am also.30. Suppose that, in addition to the three history courses previously taken, Bill also had taken and enjoyed courses in anthropology, economics, political science, and sociology. Possible answers, 35, 30, 90,64 What was one goal of the Populist Party?O A. The establishment of Grange Halls in every cityB. A lower number of immigrants allowed into the United StatesC. Getting the federal government to embrace the gold standardO D. The establishment of a national bank that would make low-interestloans to farmers. 3. Give the missing ammeter readings c and d in Figure 3.39.Assume all the bulbs are identical ones. How would you expectthe brightness of the different bulbs to compare?CA(A15 AAFigure 3.39 Use the following list of balances for Paul's Guitar Shop to answer the following question. Guitar Sales 24,800, Depreciation expense 2,000, Wage expense 10,950, Rent 500, Music Lesson Revenue 3,000, Interest expense 500, Supplies expense 500, Utilities expense 400. What is the TOTAL value of EXPENSES? * Imagine that a you and a friend are exercising together at a gym. Your friend suddenly trips and falls, and it is clear that he or she has suffered an ankle sprain. Luckily you know exactly what has happened. Explain how your friend sustained the injury and, assuming you had the necessary supplies, including a first aid kit and a phone, explain what steps could you take to stabilize your friend's injury. How much energy is transferred when the potential difference is 230 volts and the charge is 5 coulombs? A 56 kg pole vaulter falls from rest from a height of 5.1 m onto a foam rubber pad. The pole vaulter comes to rest 0.29 s after landing on the pad.Required:a. Calculate the athlete's velocity just before reaching the padb. Calculate the constant force exerted on the pole vaulter due to the collision What is the answer to this equation Please help ASAP .BCPosition (m)A 0123456789101112Time (s)The graph describes the motion of an object.The object moves withfrom A to B. Itfrom B to C. It movesfrom C to D. The measures of the angles of a triangle are shown in the figure below. Solve for x. 2. Define biomagnfication.